Convergence in distribution of Poisson variables.












1












$begingroup$


Consider ${ X_{i} }$ are independent random variables with Poisson distribution. We want to know about convergence in distribution of $frac{S_{n}-mathbb{E}(S_{n})}{sqrt{operatorname{Var}(S_{n})}}$.



There are two cases :



Case 1 : $sum_{i}lambda_{i}$ converges. Then we can say variance and mean value of $S_{n}$ converges to some fixed value. Also we know that sum of Poisson random variables is also the Poisson random variable with parameter equals sum of previous ones.
So in my opinion $frac{S_{n}-mathbb{E}(S_{n})}{sqrt{operatorname{Var}(S_{n})}}$ converges to $frac{mathrm{Pois}(c) - c}{sqrt{c}}$ , where $c = sum_{i} lambda_{i}$



Case 2 : we have that $sum_{i} lambda_{i}$ diverges. Then I guess we can try to use Central limit theorem and satisfy that $frac{S_{n}-mathbb{E}(S_{n})}{sqrt{operatorname{Var}(S_{n})}}$ converges to $N(0,1)$



Am I right ? Or where have I problems to fix ?










share|cite|improve this question











$endgroup$








  • 1




    $begingroup$
    How do you expand "i.i.d."? In standard terminology if $X_i$'s are i.i.d Poisson then the parameters $lambda_i$ are all the same, so CLT applies.
    $endgroup$
    – Kavi Rama Murthy
    May 11 '18 at 8:04










  • $begingroup$
    @KaviRamaMurthy my bad!
    $endgroup$
    – openspace
    May 11 '18 at 8:04










  • $begingroup$
    @KaviRamaMurthy changed
    $endgroup$
    – openspace
    May 11 '18 at 8:15
















1












$begingroup$


Consider ${ X_{i} }$ are independent random variables with Poisson distribution. We want to know about convergence in distribution of $frac{S_{n}-mathbb{E}(S_{n})}{sqrt{operatorname{Var}(S_{n})}}$.



There are two cases :



Case 1 : $sum_{i}lambda_{i}$ converges. Then we can say variance and mean value of $S_{n}$ converges to some fixed value. Also we know that sum of Poisson random variables is also the Poisson random variable with parameter equals sum of previous ones.
So in my opinion $frac{S_{n}-mathbb{E}(S_{n})}{sqrt{operatorname{Var}(S_{n})}}$ converges to $frac{mathrm{Pois}(c) - c}{sqrt{c}}$ , where $c = sum_{i} lambda_{i}$



Case 2 : we have that $sum_{i} lambda_{i}$ diverges. Then I guess we can try to use Central limit theorem and satisfy that $frac{S_{n}-mathbb{E}(S_{n})}{sqrt{operatorname{Var}(S_{n})}}$ converges to $N(0,1)$



Am I right ? Or where have I problems to fix ?










share|cite|improve this question











$endgroup$








  • 1




    $begingroup$
    How do you expand "i.i.d."? In standard terminology if $X_i$'s are i.i.d Poisson then the parameters $lambda_i$ are all the same, so CLT applies.
    $endgroup$
    – Kavi Rama Murthy
    May 11 '18 at 8:04










  • $begingroup$
    @KaviRamaMurthy my bad!
    $endgroup$
    – openspace
    May 11 '18 at 8:04










  • $begingroup$
    @KaviRamaMurthy changed
    $endgroup$
    – openspace
    May 11 '18 at 8:15














1












1








1





$begingroup$


Consider ${ X_{i} }$ are independent random variables with Poisson distribution. We want to know about convergence in distribution of $frac{S_{n}-mathbb{E}(S_{n})}{sqrt{operatorname{Var}(S_{n})}}$.



There are two cases :



Case 1 : $sum_{i}lambda_{i}$ converges. Then we can say variance and mean value of $S_{n}$ converges to some fixed value. Also we know that sum of Poisson random variables is also the Poisson random variable with parameter equals sum of previous ones.
So in my opinion $frac{S_{n}-mathbb{E}(S_{n})}{sqrt{operatorname{Var}(S_{n})}}$ converges to $frac{mathrm{Pois}(c) - c}{sqrt{c}}$ , where $c = sum_{i} lambda_{i}$



Case 2 : we have that $sum_{i} lambda_{i}$ diverges. Then I guess we can try to use Central limit theorem and satisfy that $frac{S_{n}-mathbb{E}(S_{n})}{sqrt{operatorname{Var}(S_{n})}}$ converges to $N(0,1)$



Am I right ? Or where have I problems to fix ?










share|cite|improve this question











$endgroup$




Consider ${ X_{i} }$ are independent random variables with Poisson distribution. We want to know about convergence in distribution of $frac{S_{n}-mathbb{E}(S_{n})}{sqrt{operatorname{Var}(S_{n})}}$.



There are two cases :



Case 1 : $sum_{i}lambda_{i}$ converges. Then we can say variance and mean value of $S_{n}$ converges to some fixed value. Also we know that sum of Poisson random variables is also the Poisson random variable with parameter equals sum of previous ones.
So in my opinion $frac{S_{n}-mathbb{E}(S_{n})}{sqrt{operatorname{Var}(S_{n})}}$ converges to $frac{mathrm{Pois}(c) - c}{sqrt{c}}$ , where $c = sum_{i} lambda_{i}$



Case 2 : we have that $sum_{i} lambda_{i}$ diverges. Then I guess we can try to use Central limit theorem and satisfy that $frac{S_{n}-mathbb{E}(S_{n})}{sqrt{operatorname{Var}(S_{n})}}$ converges to $N(0,1)$



Am I right ? Or where have I problems to fix ?







probability-theory proof-verification weak-convergence poisson-distribution probability-limit-theorems






share|cite|improve this question















share|cite|improve this question













share|cite|improve this question




share|cite|improve this question








edited Nov 24 '18 at 22:52









Davide Giraudo

125k16150261




125k16150261










asked May 11 '18 at 7:58









openspaceopenspace

3,4352822




3,4352822








  • 1




    $begingroup$
    How do you expand "i.i.d."? In standard terminology if $X_i$'s are i.i.d Poisson then the parameters $lambda_i$ are all the same, so CLT applies.
    $endgroup$
    – Kavi Rama Murthy
    May 11 '18 at 8:04










  • $begingroup$
    @KaviRamaMurthy my bad!
    $endgroup$
    – openspace
    May 11 '18 at 8:04










  • $begingroup$
    @KaviRamaMurthy changed
    $endgroup$
    – openspace
    May 11 '18 at 8:15














  • 1




    $begingroup$
    How do you expand "i.i.d."? In standard terminology if $X_i$'s are i.i.d Poisson then the parameters $lambda_i$ are all the same, so CLT applies.
    $endgroup$
    – Kavi Rama Murthy
    May 11 '18 at 8:04










  • $begingroup$
    @KaviRamaMurthy my bad!
    $endgroup$
    – openspace
    May 11 '18 at 8:04










  • $begingroup$
    @KaviRamaMurthy changed
    $endgroup$
    – openspace
    May 11 '18 at 8:15








1




1




$begingroup$
How do you expand "i.i.d."? In standard terminology if $X_i$'s are i.i.d Poisson then the parameters $lambda_i$ are all the same, so CLT applies.
$endgroup$
– Kavi Rama Murthy
May 11 '18 at 8:04




$begingroup$
How do you expand "i.i.d."? In standard terminology if $X_i$'s are i.i.d Poisson then the parameters $lambda_i$ are all the same, so CLT applies.
$endgroup$
– Kavi Rama Murthy
May 11 '18 at 8:04












$begingroup$
@KaviRamaMurthy my bad!
$endgroup$
– openspace
May 11 '18 at 8:04




$begingroup$
@KaviRamaMurthy my bad!
$endgroup$
– openspace
May 11 '18 at 8:04












$begingroup$
@KaviRamaMurthy changed
$endgroup$
– openspace
May 11 '18 at 8:15




$begingroup$
@KaviRamaMurthy changed
$endgroup$
– openspace
May 11 '18 at 8:15










1 Answer
1






active

oldest

votes


















0












$begingroup$

The answer is easy if you compute the characteristic function explicitly. Let $u_n=lambda_1+lambda_2++...+lambda_n$. Then $Ee^{it(S_n-ES_n)/sqrt (Var(S_n)}=e^{-itsqrt (u_n)}prod_{j=1}^{n}(Ee^{itX_j /sqrt(u_n)}$. The Poisson characteristic function (paramater $lambda $) is $e^{-lambda (1- e^{it})}$. We get $e^{-itsqrt (u_n)}e^{-u_n (1-e^{it/sqrt (u_n)})}$. Using the Taylor expansion of $e^{it/sqrt (u_n)}$ up to the term in $t^{2}$ you will see that that characteristic function indeed converges to $e^{-t^{2}/2}$ if $sum lambda_j =infty$. The characteristic function also converges when $sum lambda_j <infty$ and you can write down the limiting characteristic function explictly.






share|cite|improve this answer









$endgroup$













    Your Answer





    StackExchange.ifUsing("editor", function () {
    return StackExchange.using("mathjaxEditing", function () {
    StackExchange.MarkdownEditor.creationCallbacks.add(function (editor, postfix) {
    StackExchange.mathjaxEditing.prepareWmdForMathJax(editor, postfix, [["$", "$"], ["\\(","\\)"]]);
    });
    });
    }, "mathjax-editing");

    StackExchange.ready(function() {
    var channelOptions = {
    tags: "".split(" "),
    id: "69"
    };
    initTagRenderer("".split(" "), "".split(" "), channelOptions);

    StackExchange.using("externalEditor", function() {
    // Have to fire editor after snippets, if snippets enabled
    if (StackExchange.settings.snippets.snippetsEnabled) {
    StackExchange.using("snippets", function() {
    createEditor();
    });
    }
    else {
    createEditor();
    }
    });

    function createEditor() {
    StackExchange.prepareEditor({
    heartbeatType: 'answer',
    autoActivateHeartbeat: false,
    convertImagesToLinks: true,
    noModals: true,
    showLowRepImageUploadWarning: true,
    reputationToPostImages: 10,
    bindNavPrevention: true,
    postfix: "",
    imageUploader: {
    brandingHtml: "Powered by u003ca class="icon-imgur-white" href="https://imgur.com/"u003eu003c/au003e",
    contentPolicyHtml: "User contributions licensed under u003ca href="https://creativecommons.org/licenses/by-sa/3.0/"u003ecc by-sa 3.0 with attribution requiredu003c/au003e u003ca href="https://stackoverflow.com/legal/content-policy"u003e(content policy)u003c/au003e",
    allowUrls: true
    },
    noCode: true, onDemand: true,
    discardSelector: ".discard-answer"
    ,immediatelyShowMarkdownHelp:true
    });


    }
    });














    draft saved

    draft discarded


















    StackExchange.ready(
    function () {
    StackExchange.openid.initPostLogin('.new-post-login', 'https%3a%2f%2fmath.stackexchange.com%2fquestions%2f2776301%2fconvergence-in-distribution-of-poisson-variables%23new-answer', 'question_page');
    }
    );

    Post as a guest















    Required, but never shown

























    1 Answer
    1






    active

    oldest

    votes








    1 Answer
    1






    active

    oldest

    votes









    active

    oldest

    votes






    active

    oldest

    votes









    0












    $begingroup$

    The answer is easy if you compute the characteristic function explicitly. Let $u_n=lambda_1+lambda_2++...+lambda_n$. Then $Ee^{it(S_n-ES_n)/sqrt (Var(S_n)}=e^{-itsqrt (u_n)}prod_{j=1}^{n}(Ee^{itX_j /sqrt(u_n)}$. The Poisson characteristic function (paramater $lambda $) is $e^{-lambda (1- e^{it})}$. We get $e^{-itsqrt (u_n)}e^{-u_n (1-e^{it/sqrt (u_n)})}$. Using the Taylor expansion of $e^{it/sqrt (u_n)}$ up to the term in $t^{2}$ you will see that that characteristic function indeed converges to $e^{-t^{2}/2}$ if $sum lambda_j =infty$. The characteristic function also converges when $sum lambda_j <infty$ and you can write down the limiting characteristic function explictly.






    share|cite|improve this answer









    $endgroup$


















      0












      $begingroup$

      The answer is easy if you compute the characteristic function explicitly. Let $u_n=lambda_1+lambda_2++...+lambda_n$. Then $Ee^{it(S_n-ES_n)/sqrt (Var(S_n)}=e^{-itsqrt (u_n)}prod_{j=1}^{n}(Ee^{itX_j /sqrt(u_n)}$. The Poisson characteristic function (paramater $lambda $) is $e^{-lambda (1- e^{it})}$. We get $e^{-itsqrt (u_n)}e^{-u_n (1-e^{it/sqrt (u_n)})}$. Using the Taylor expansion of $e^{it/sqrt (u_n)}$ up to the term in $t^{2}$ you will see that that characteristic function indeed converges to $e^{-t^{2}/2}$ if $sum lambda_j =infty$. The characteristic function also converges when $sum lambda_j <infty$ and you can write down the limiting characteristic function explictly.






      share|cite|improve this answer









      $endgroup$
















        0












        0








        0





        $begingroup$

        The answer is easy if you compute the characteristic function explicitly. Let $u_n=lambda_1+lambda_2++...+lambda_n$. Then $Ee^{it(S_n-ES_n)/sqrt (Var(S_n)}=e^{-itsqrt (u_n)}prod_{j=1}^{n}(Ee^{itX_j /sqrt(u_n)}$. The Poisson characteristic function (paramater $lambda $) is $e^{-lambda (1- e^{it})}$. We get $e^{-itsqrt (u_n)}e^{-u_n (1-e^{it/sqrt (u_n)})}$. Using the Taylor expansion of $e^{it/sqrt (u_n)}$ up to the term in $t^{2}$ you will see that that characteristic function indeed converges to $e^{-t^{2}/2}$ if $sum lambda_j =infty$. The characteristic function also converges when $sum lambda_j <infty$ and you can write down the limiting characteristic function explictly.






        share|cite|improve this answer









        $endgroup$



        The answer is easy if you compute the characteristic function explicitly. Let $u_n=lambda_1+lambda_2++...+lambda_n$. Then $Ee^{it(S_n-ES_n)/sqrt (Var(S_n)}=e^{-itsqrt (u_n)}prod_{j=1}^{n}(Ee^{itX_j /sqrt(u_n)}$. The Poisson characteristic function (paramater $lambda $) is $e^{-lambda (1- e^{it})}$. We get $e^{-itsqrt (u_n)}e^{-u_n (1-e^{it/sqrt (u_n)})}$. Using the Taylor expansion of $e^{it/sqrt (u_n)}$ up to the term in $t^{2}$ you will see that that characteristic function indeed converges to $e^{-t^{2}/2}$ if $sum lambda_j =infty$. The characteristic function also converges when $sum lambda_j <infty$ and you can write down the limiting characteristic function explictly.







        share|cite|improve this answer












        share|cite|improve this answer



        share|cite|improve this answer










        answered May 11 '18 at 9:22









        Kavi Rama MurthyKavi Rama Murthy

        53.6k32055




        53.6k32055






























            draft saved

            draft discarded




















































            Thanks for contributing an answer to Mathematics Stack Exchange!


            • Please be sure to answer the question. Provide details and share your research!

            But avoid



            • Asking for help, clarification, or responding to other answers.

            • Making statements based on opinion; back them up with references or personal experience.


            Use MathJax to format equations. MathJax reference.


            To learn more, see our tips on writing great answers.




            draft saved


            draft discarded














            StackExchange.ready(
            function () {
            StackExchange.openid.initPostLogin('.new-post-login', 'https%3a%2f%2fmath.stackexchange.com%2fquestions%2f2776301%2fconvergence-in-distribution-of-poisson-variables%23new-answer', 'question_page');
            }
            );

            Post as a guest















            Required, but never shown





















































            Required, but never shown














            Required, but never shown












            Required, but never shown







            Required, but never shown

































            Required, but never shown














            Required, but never shown












            Required, but never shown







            Required, but never shown







            Popular posts from this blog

            Biblatex bibliography style without URLs when DOI exists (in Overleaf with Zotero bibliography)

            How to change which sound is reproduced for terminal bell?

            Can I use Tabulator js library in my java Spring + Thymeleaf project?